PT 61 LR 1 Q 22 - Strengthen X: Premise Booster Forum

Prepare for the LSAT or discuss it with others in this forum.
Post Reply
TTX

New
Posts: 26
Joined: Thu Jan 03, 2013 8:00 am

PT 61 LR 1 Q 22 - Strengthen X: Premise Booster

Post by TTX » Wed May 29, 2013 1:39 pm

PT 61 LR 1 Q 22

Psychiatrist: In treating first-year...

This is a strengthen EXCEPT question, so all the wrong answer choices strength the the argument in certain way.

Since answer choice (D) is one of the wrong answers, it must strengthen the argument. I understand it strengthens the argument by addressing one of the unstated premises, which is that the standard screening instruments used are not faulty. But to me this looks awfully close to a premise booster, and we know premise boosters do not strengthen an argument.

So can someone explain why answer choice (D) isn't a premise booster?

User avatar
CardozoLaw09

Gold
Posts: 2232
Joined: Sat Aug 28, 2010 1:58 pm

Re: PT 61 LR 1 Q 22 - Strengthen X: Premise Booster

Post by CardozoLaw09 » Wed May 29, 2013 1:55 pm

Saying the instruments are "extremely accurate" isn't really boosting the premise. We can assume the instruments used produce accurate results, but by saying they're "extremely" accurate goes further in substantiating the conclusion.

Daily_Double

Silver
Posts: 1031
Joined: Tue Dec 04, 2012 8:45 pm

Re: PT 61 LR 1 Q 22 - Strengthen X: Premise Booster

Post by Daily_Double » Wed May 29, 2013 2:04 pm

False. Premise boosters do strengthen an argument, however, they do not strengthen the argument's core. By strengthening a premise, the argument builds support for its conclusion. Unfortunately, when we are looking to strengthen/weaken an argument we usually focus upon the core so these answers are often overlooked and usually incorrect.

Let's look at your question. So here we have a conclusion, 1st year students with a specific characteristic could reduce spending without increasing undesirable mental effects. Why? Because those who spend less score the same on a test for these undesirable mental effects as people who spend more. Ok, so what the argument is concluding, on the basis of a correlation, is that the level of spending does not influence these undesirable mental effects. It is also assuming these tests are a reliable indicator of the levels of these undesirable mental effects.

So now we understand the argument, let's look at answer D. This strengthens the support for the argument, we expected this earlier. By strengthening the accuracy of the test, the argument strengthens the correlation, thus strengthening the conclusion.

Hope this helps.

Post Reply

Return to “LSAT Prep and Discussion Forum”